GU K Questions

Ace your homework & exams now with Quizwiz!

A 23-year-old African American man with AIDS is sent for workup of nephrotic syndrome. His blood pressure is 140/82 mm Hg. He has 3+ pitting edema in both legs. His risk factor for AIDS is IV heroin use. His creatinine level is 2.0 mg/dL, and urine reveals 3+ protein, no blood. A kidney biopsy specimen would most likely reveal which of the following? A. Diabetic nephropathy B. Focal segmental glomerular sclerosis C. IgA nephropathy D. Membranous nephropathy E. Minimal change disease (Nil disease)

B. Focal segmental glomerular sclerosis Focal segmental glomerular sclerosis(FSGS) is the type of nephropathy most commonly seen in African American IV drug users who have AIDS. It is likely to lead to a very rapid loss of renal function.

A 5-month-old boy is brought to the office for a mass in his left groin area. The infant is on the examination table, quietly sucking on his pacifier. On examination there is a sacculated cavity that does not reach the inguinal ring. Light easily passes through the sac. Which of the following is the most likely diagnosis? A. Hematoma B. Hydrocele C. Inguinal hernia D. Testicular torsion E. Testicular tumor

B. Hydrocele A hydrocele is a fluid-filled sac in the scrotum that is nonpainful and transilluminates on examination.

A 3-week-old boy is brought to a health care provider with a 1-week history of forceful vomiting. He has been vomiting after almost every feeding. The vomitus contains mostly undigested formula and is nonbilious. On examination his oral mucosa is dry, his anterior fontanelle appears to be depressed and his capillary refill is 3-4 seconds. Abdominal examination reveals a 2-cm mass in the epigastrium. The remainder of the examination is unremarkable. Which of the following electrolyte findings will most likely be seen? A. Hypochloremic metabolic acidosis B. Hypochloremic metabolic alkalosis C. Normal electrolytes D. Respiratory acidosis with metabolic compensation E. Respiratory alkalosis

B. Hypochloremic metabolic alkalosis This infant most likely has pyloric stenosis. Pyloric stenosis occurs in approximately 1:500 births, with male infants > female infants. The incidence is far greater in full-term infants than in preterm infants, and first-born children are also more likely to be affected. Clinical manifestations include projectile nonbilious vomiting shortly after feeding and an olive-sized mass palpable in the epigastrium. Symptoms typically present in the 2nd or 3rd week of life. With continued vomiting, hydrogen ions and chloride ions are lost, causing a hypochloremic metabolic alkalosis. The infant might also be hypokalemic from repeated vomiting. Plain radiographs may demonstrate the absence of air distal to the obstructed pylorus. In barium contrast studies, a small amount of barium may pass through the hypertrophied pylorus, causing the "string sign." Ultrasound studies are also useful in demonstrating the hypertrophied pylorus. The dehydration and electrolyte abnormalities should be corrected. Definite treatment is pyloromyotomy.

A man with prostatic hyperplasia is placed on terazosin (Minipres). To which of the following classes of medications does this agent belong? A. Alpha-blockers B. Gonadotropin-releasing hormone inhibitors C. Direct testosterone antagonists D. 5-Alpha reductase inhibitors E. Phosphodiesterase inhibitors

A. Alpha-blockers Terazosin is an alpha-1 adrenergic blocker that works by relaxing the smooth muscle of the bladder neck and increases urine flow.

A 65-year-old woman is admitted to the hospital for constant, severe abdominal pain that has worsened over the previous week. She has no other associated symptoms, such as nausea or vomiting, but has noticed that her daily urine output has sharply decreased. She has had a constant desire to urinate, but when she tries, only a small amount of bloody urine is discharged. The patient is a long-time smoker, having smoked 3 packs per day for 45+ years, though she claims to have quit 2 days ago. Bladder ultrasound in the emergency department reveals a large mass consistent with bladder cancer, as well as significant urinary retention. Which of the following is most likely to be detected upon imaging the patient's genitourinary system? A. Bilateral hydronephrosis B. Bladder dilation C. Bladder dyskinesis D. Unilateral hydronephrosis E. Ureteral dilation

A. Bilateral hydronephrosis Urinary retention is most often caused by an anatomic obstruction to urine outflow. In men, this is often caused by benign prostatic hyperplasia (BPH). Women have a variety of causes. The common manifestation of prolonged urinary retention is bilateral hydronephrosis caused by urinary retention and pressure increases in the urinary system. A large bladder mass would compress both distal ureters, leading to inability of the kidneys to empty with resultant bilateral hydronephrosis.

A 45-year-old woman is admitted to the hospital after being diagnosed with post-streptococcal glomerulonephritis. The patient was well until 3 weeks ago, when she noticed severe body swelling and blood in her urine. Two weeks prior to that she had a severe sore throat and high fever. She sought no treatment at that time. She finally presented to the emergency department because her shoes no longer fit. On admission her creatinine level was 6.6 mg/dL, and she was noted to have 1 g of protein on 24-h urine collection. The patient had a dialysis catheter placed, and hemodialysis was started on hospital day 3 for hyperkalemia. Which of the following dietary supplements or regimens will most likely benefit this patient? A. Calcium supplementation B. High-protein diet C. Low-fat diet D. Vitamin C supplementation E. Vitamin E supplementation

A. Calcium supplementation Renal failure wreaks havoc on a number of metabolic functions. One of these is calcium homeostasis. Renal failure patients absorb less calcium and develop secondary hyperparathyroidism in an attempt to correct the hypocalcemia. This leads to bone calcium deficits and pathologic bone lesions. Calcium supplementation slows this process. All patients who have chronic renal failure should have calcium supplementation unless there is a contraindication for its use.

A 67-year-old man comes to the emergency department with abdominal and back pain together with an inability to urinate for 12 hours. Over the last 2 years he has had progressive urinary hesitancy and increased urine "dribbling." Also, his urine stream has become progressively weaker. His temperature is 37.0° C (98.6° F). Mucous membranes are moist. Heart is tachycardic and regular. Lungs are clear. Abdomen is soft and diffusely tender. Bladder is distended and tense. There is no edema in the lower extremities. Rectal examination reveals a boggy, large prostate. There is trace blood in the stool. Laboratory studies show: Sodium 146 mEq/L Potassium 5.2 mEq/L Chloride 100 mEq/L Bicarbonate 23 mEq/L Glucose 106 mg/dL BUN 77 mg/dL Creatinine 3.3 mg/dL Which of the following is the most appropriate next step in management? A. Catheter placement B. Emergent transurethral resection of the prostate gland C. Fluid challenge D. Furosemide challenge E. Renal ultrasound

A. Catheter placement This patient has postrenal azotemia. The most likely etiology of this condition in an otherwise healthy man is prostate hypertrophy. Other etiologies include obstructing neoplasm, clot in the ureter, and strictures. Our patient has no urine output, a distended bladder, an enlarged prostate, and a classic history for progressive benign prostatic hyperplasia. Placement of a urinary catheter would likely relieve this obstruction and therefore relieve his symptoms. We would expect that his creatinine level would begin to come down quickly as this obstruction is relieved.

A 28-year-old man decides to donate a kidney to his brother, who has chronic kidney disease (CKD), after HLA typing suggests that he would be a suitable donor. Which of the following renal indices would be expected to be decreased in the donor after full recovery from the operation? A. Creatinine clearance B. Creatinine production C. Daily excretion of sodium D. Plasma creatinine concentration E. Renal excretion of creatinine

A. Creatinine clearance Because creatinine is freely filtered by the glomerulus, but not secreted or reabsorbed to a significant extent, the renal clearance of creatinine is approximately equal to the glomerular filtration rate. In fact, creatinine clearance is commonly used to assess renal function in the clinical setting. When a kidney is removed, the total glomerular filtration rate decreases, because 50% of the nephrons have been removed, which causes the creatinine clearance to decrease. In turn, the plasma creatinine concentration (choice D)increases until the rate of creatinine excretion by the kidneys (choice E) is equal to the rate of creatinine production by the body. Recall that creatinine excretion = GFR × plasma creatinine concentration. Creatinine excretion therefore is normal when GFR is decreased following removal of a kidney, because the plasma concentration of creatinine is elevated. Creatinine is a waste product of metabolism.

A 29-year-old man comes to the clinic with a 2-day history of severe left-sided scrotal pain and swelling. He is sexually active and has multiple sexual partners. He has no history of sexually transmitted diseases. His temperature is 38.2° C (100.8° F), blood pressure 120/70 mm Hg, and pulse 80/min. Examination shows unilateral intra-scrotal tenderness and swelling. Raising the testicle makes the pain less intense. Which of the following is the most likely diagnosis? A. Epididymitis B. Prostatitis C. Testicular torsion D. Urethritis E. Varicocele

A. Epididymitis This patient has epididymitis, most likely caused by Chlamydia trachomatis. Epididymitis refers to inflammation of the epididymis, which leads to unilateral intra-scrotal pain, swelling, and fever. Elevation of the affected testis usually relieves the pain to some extent (Prehn sign). Asymptomatic urethritis can be associated with epididymitis caused by C. trachomatis or Neisseria gonorrhoeae. Urinalysis may reveal pyuria. Urethral culture, urine polymerase chain reaction (PCR), or ligase chain reaction (LCR) can confirm the diagnosis. Treatment is azithromycin, doxycycline, or tetracycline. The sexual partners should be treated to avoid reinfection.

A 25-year-old woman comes to the health care provider because of pain and burning with urination. She states that the symptoms started 2 days ago and have worsened since. She has no fever or chills and has never had these symptoms before. She has hypothyroidism for which she takes thyroid hormone replacement; otherwise she has no medical problems. Her temperature is 37° C (98.6° F). Examination is unremarkable, including a normal pelvic examination. KOH and normal saline "wet prep" is performed on her vaginal discharge and is negative. Urinalysis reveals numerous white blood cells. Which of the following is the most likely pathogen? A. Escherichia coli B. Neisseria gonorrhoeae C. Pseudomonas species D. Staphylococcus saprophyticus E. Trichomonas vaginalis

A. Escherichia coli This patient has findings that are most consistent with a lower urinary tract infection (UTI). A lower UTI refers to infection of the bladder (cystitis) or urethra (urethritis). The principal complaints for women who have lower UTIs are dysuria, urgency, and frequency. Most often examination will be unremarkable. Occasionally, suprapubic tenderness may be present. A urinalysis will often reveal a positive leukocyte esterase or nitrite test. The microscopic analysis will show white blood cells. The most significant risk factors are related to sexual activity and hypoestrogenism. These factors lead to invasion by pathogenic organisms. E. coli is by far the most common causative organism in cases of acute uncomplicated cystitis. It is responsible for approximately 80% of these cases. Trimethoprim sulfamethoxazole is the treatment of choice for uncomplicated UTIs. On the other hand, asymptomatic bacteriuria is only treated in pregnant patients to decrease the risk for pyelonephritis.

A 64-year-old man presents to his health care provider complaining of difficulty urinating and dribbling at the end of urination. Which of the following drugs would be most appropriate for treating this man's condition? A. Finasteride (Proscar) B. Leuprolide (Lupron) C. Mifepristone (Mifeprex, RU-486) D. Cabergoline (Dostinex) E. Tamoxifen (Nolvadex)

A. Finasteride (Proscar) Finasteride, an inhibitor of 5-alpha-reductase, prevents the conversion of testosterone to dihydrotestosterone (DHT). Because dihydrotestosterone is essential for the normal growth and development of the prostate gland, finasteride is an effective treatment for benign prostatic hyperplasia, which is a DHT-dependent process.

A 45-year-old white man has had a 16-pound weight gain together with massive lower extremity edema and fatigue that has been progressively worsening over the last 2 weeks. His urine is normal colored but foamy. He denies any recent skin or throat infection, diabetes, carpal tunnel syndrome, or symptoms of heart failure. His history reveals that he has long-standing, untreated hepatitis C infection. His temperature is 37.0°C (98.6°F), blood pressure 170/70 mm Hg, pulse 93/min, and respirations 18/min. Physical examination reveals massive upper and lower extremity edema along with sacral edema. His lungs have decreased breath sounds at the bases bilaterally. His heart and abdominal examination are within normal limits. He has no retinopathy. Laboratory studies show: Sodium 130 mEq/L Potassium 4.5 mEq/L Chloride 100 mEq/L Bicarbonate 24 mEq/L Glucose 116 mg/dL BUN 27 mg/dL Creatinine 1.7 mg/dL Albumin 1.7 g/dL LDL cholesterol 450 mg/dL Hemoglobin 12 mg/dL White blood cells 7700/mm^3 Platelets 270,000mm^3 Hematocrit 31% Urinalysis: Color Clear Leukocyte esterase negative Nitrite negative Protein 4+ Blood negative 24-hour urine protein collection 8.3 g/24 hours A. Glomeruli slightly enlarged, normal cellularity, thickened capillary walls and basement membrane B. Glomerulosclerosis in some glomeruli with other glomeruli appearing normal C. Kimmelstiel-Wilson lesions D. Normal appearance with light microscopy E. Patchy and irregular interstitial infiltration with inflammatory cells

A. Glomeruli slightly enlarged, normal cellularity, thickened capillary walls and basement membrane This patient most likely has membranous nephropathy. A biopsy specimen would most likely show slightly enlarged glomeruli, normal cellularity, and thickened capillary walls and basement membrane. Membranous nephropathy is the most common cause of non-diabetic nephrotic syndrome in Caucasian adults. Risk factors for this condition include male sex, age over 40, and previous history of hepatitis B or C.

A 35-year-old man with end-stage renal disease presents with syncope. The patient denies chest pain or dyspnea. Physical examination is remarkable for a left-arm AV fistula. His creatinine level is 14 mg/dL, urea nitrogen 88 mg/dL, and potassium 8.8 mEq/L. Electrocardiogram reveals sinus bradycardia with a sine wave pattern. Which of the following is the most appropriate initial step in management? A. IV calcium gluconate B. IV dextrose C. IV furosemide D. IV sodium bicarbonate E. Rectal sodium polystyrene sulfonate

A. IV calcium gluconate IV calcium gluconate will work very rapidly to counteract the effect of the high potassium on the heart and muscle and should be given first. The calcium will work to stabilize the heart and will lower the risk for malignant arrhythmia from occurring. This treatment will not actually lower the level of potassium, however. All of the other agents are good at lowering potassium (choices B, C D, and E), but do not have the same immediate effect to protect the heart that calcium has. These agents therefore should be given after IV calcium when there are severe ECG changes.

A 20-year-old woman presents with a 2-day history of dysuria and increased urinary frequency. She states that she was recently sexually active. Physical examination reveals a temperature of 38.2° C (100.7° F) with normal vital signs. Gynecologic examination reveals no evidence of discharge, vaginitis, or cervicitis. Urinalysis reveals 14 WBC/hpf with many gram-negative rods. Which of the following is the most appropriate pharmacotherapy? A. Sulfamethoxazole/trimethoprim (Bactrim) B. Ceftriaxone (Rocephin) C. Fluconazole (Diflucan) D. Gentamicin E. Metronidazole (Flagyl)

A. Sulfamethoxazole/trimethoprim (Bactrim) The patient's presentation is consistent with a simple urinary tract infection (UTI); there is a short history of dysuria, increased urinary frequency, and the appearance of white blood cells and gram-negative rods in the urine. UTIs are common in women after sexual activity. The infection is likely caused by urethral trauma during intercourse, which leads to bacterial contamination of the bladder. Most of these infections are caused by Escherichia coli (a gram-negative rod). For uncomplicated UTI, the most appropriate therapy is a short 3-day course of sulfamethoxazole/trimethoprim.

A severely ill patient in shock develops acute renal failure with oliguria and azotemia. The urine osmolality approaches that of the glomerular ultrafiltrate (specific gravity 1.010). The urine is brown. The microscopic examination of the patient's urine reveals renal epithelial cell casts. A renal biopsy would most likely show which of the following? A. Acute pyelonephritis B. Acute tubular necrosis C. Chronic pyelonephritis D. Crescentic glomerulonephritis E. Renal cell carcinoma

B. Acute tubular necrosis In this typical clinical scenario, acute tubular necrosis develops secondary to shock and further destabilizes an already desperately ill patient. Acute tubular necrosis can also occur following exposure to nephrotoxins or as a manifestation of acute rejection in a transplanted kidney. The prognosis depends on the patient's overall health. In patients who recover, the initial oliguric phase (lasting 1 to 2 weeks) is followed by a diuretic phase (weeks to months) before eventual restoration of tubular function.

A 27-year-old man is admitted to the hospital for lethargy and confusion. The patient has a 5-year history of severe hypertension that has been refractory to medical therapy. Over the past few days, the man has become increasingly lethargic and confused. On the day of admission, he was found in his home, lying on the floor and nonresponsive. His roommate called EMS, and the patient was brought emergently to the hospital where he was found to have hypotension, dry mucous membranes, and low jugular venous pressure. Laboratory values are as follows: Serum sodium 112 mEq/L Serum potassium 5.1 mEq/L Serum osmolality 230 mOsmol/kg H2O Urea nitrogen (BUN) 4 mEq/L Which of the following substances is most likely mediating the hyponatremia? A. Aldosterone B. Antidiuretic hormone (ADH) C. Cortisol D. Somatostatin E. Thyroid-stimulating hormone (TSH)

B. Antidiuretic hormone (ADH) Hyponatremia is the most common electrolyte abnormality encountered in the hospital. The combination of hypotension, dry mucous membranes, low jugular venous pressure, and low urine sodium suggests that the patient is markedly volume depleted. In states in which the body is volume depleted, the drive to replenish intravascular volume is greater than the need to maintain osmolality. As such, secretion of antidiuretic hormone (ADH) from the pituitary increases, and free water is retained at the level of the collecting ducts in the kidney. This process can lead to profound hyponatremia, as occurred in this case.

A 33-year-old man receiving chemotherapy for testicular carcinoma develops signs of renal tubular damage. Which of the following drugs is most likely responsible for this nephrotoxicity? A. Bleomycin (Blenoxane) B. Cisplatin (Platinol) C. Cyclophosphamide (Cytoxan) D. Vinblastine E. Vincristine (Oncovin)

B. Cisplatin (Platinol) Cisplatin is an antineoplastic drug used in the treatment of carcinoma of the testes (along with bleomycin and vinblastine), ovaries, bladder, breast, and lung (especially small cell). Along with the typical side effects of nausea, vomiting, and bone marrow suppression, cisplatin is notable for its dose-limiting nephrotoxicity and ototoxicity.

A patient with renal cell carcinoma is scheduled for surgery. The physician assistant is assisting a surgeon who is planning to approach the suprarenal area by removing the 12th rib. With this approach, the surgeon should take particular care to avoid damaging which of the following structures? A. Aorta B. Diaphragm C. Pancreas D. Stomach E. Vena cava

B. Diaphragm The diaphragm is at particular risk because it is closely related to both the 12th rib and kidney. Surgeons sometimes call this the "perilous pleura" because of the care they must exercise with this approach. The close physical relationship also accounts for the pleural effusions seen in some patients with kidney infections. The other structures listed in the answers are not particularly vulnerable during the suggested surgical approach.

A 54-year-old woman comes to the emergency department with severe left-sided flank pain that is referred to the left labia majora. She is very uncomfortable lying still on the stretcher and is continuously shifting, trying to find a comfortable position. A nonenhanced helical CT scan shows a 10-mm ureteral stone at the ureteropelvic junction. She has a normal coagulation profile. Which of the following would most likely be the best therapy in this case? A. Plenty of fluids and analgesics and await spontaneous passage B. Extracorporeal shock wave lithotripsy (ESWL) C. Endoscopic retrograde basket extraction D. Endoscopic retrograde laser vaporization of the stone E. Open surgical removal

B. Extracorporeal shock wave lithotripsy (ESWL) Extracorporeal shock wave lithotripsy (ESWL) is the most commonly used method to fragment urinary stones and allow their passage. Pregnancy and coagulation problems are contraindications. The first one is ruled out by her age; the second one we have been told she does not have. Because the stone is 10 mm, it is less likely to pass on its own because of its large size. When ESWL is performed, the patient will also have placement of ureteral stent in order to allow the fragments of stone to pass following this procedure.

A 74-year-old man has not been able to pass urine today, but had been able to do so normally the previous 2 days. Physical examination is remarkable for a blood pressure of 175/90 mm Hg. Laboratory examination reveals a serum creatinine level of 4.5 mg/dL and a blood urea nitrogen level of 115 mg/dL. Urinalysis reveals a specific gravity of 1.010 mg/dL and an occasional white blood cell per high-powered field. Which of the following medications is indicated at this time? A. Benazepril (Lotensin) B. Doxazosin (Cardura) C. Furosemide (Lasix) D. Hyoscyamine (Levsin) E. Phenazopyridine (Pyridium, Azo)

B. Doxazosin (Cardura) Prostate hypertrophy in elderly men is very common; therefore, it should be considered as a primary cause of renal failure until proven otherwise. The patient's signs and symptoms are consistent with obstructive uropathy; there is a history of high urine output followed by periods of almost no urine output. This pattern leads to the accumulation of urine in the collecting system, which creates a high-pressure system. The high pressure is then transmitted back to the kidney and results in reduced GFR. Because the patient's obstructive uropathy is most likely caused by prostate hypertrophy, doxazosinshould be used to treat the cause of these signs and symptoms. Doxazosin is a peripherally acting alpha-1-adrenergic blocking agent indicated for the treatment of urinary outflow obstruction secondary to benign prostatic hyperplasia (BPH). It is also indicated for the treatment of hypertension, especially in men who have BPH. The use of this agent therefore will correct the obstructive uropathy and treat his hypertension.

A 28-year-old man with chronic kidney disease comes to the clinic for a checkup. He feels well except for some mild fatigue with exercise. He has creatinine level 6.7 mg/dL, urea nitrogen level 70 mg/dL, and hemoglobin 8.4 g/dL. Platelet count is 200,000/mm3. Blood pressure is 140/80 mm Hg and lungs are clear. Heart is regular with a soft systolic murmur. Abdomen is soft. Stool is guaiac-negative for occult blood. Ferritin level is 200 ng/mL (normal 10-200 ng/mL) and total iron binding capacity is 300 (normal 250-420). MCV is 85 fL. Which of the following would be the most appropriate treatment for his anemia? A. No treatment, follow values B. Erythropoietin C. Folic acid D. Trial of iron therapy E. Blood transfusion

B. Erythropoietin The patient has an anemia of advanced chronic kidney disease. It should not be simply watched (choice A), as hemoglobin this low can cause a high output state with cardiac hypertrophy. The patient should be started on erythropoietin by the subcutaneous route. The patient's other lab tests support adequate iron reserves to allow the erythropoietin to stimulate reticulocytosis.

A 20-year-old man with a history of nephrotic syndrome is seen in clinic after a renal biopsy. He presented a week ago with generalized swelling and foamy urine without hematuria.His blood pressure is 100/60 mm Hg. Examination reveals periorbital edema, with clear lungs, a normal cardiac examination, and 4+ edema and anasarca. His 24-hour urine collection contains 8 grams of protein and serum creatinine is 0.8 mg/dL. Urinalysis shows 4+ protein, but no red blood cells or casts. Electron microscopic evaluation of a renal biopsy specimen shows that the basement membrane of the glomeruli is morphologically normal, with no electron-dense material deposition. In the visceral epithelial cells, there is a uniform and diffuse effacement of foot processes. Which of the following is the most appropriate next step in management? A. IV pulse cyclophosphamide B. Oral prednisone C. Observation D. Oral FK506 (Prograf) E. Renal transplant

B. Oral prednisone Minimal change disease is a cause of nephrotic syndrome. Although it is more common in children, it can still be seen in adults. These patients will usually have a paucity of hematuria or azotemia. Most patients will have a good response to oral steroids, which are considered to be the initial treatment of choice.

A 7-day-old boy who is the product of an uncomplicated gestation is brought to the health care provider because of hypospadias. The infant is otherwise healthy and is urinating without any difficulty. On physical examination, vital signs are stable, lungs are clear, and the heart is beating at a regular rate. The only abnormal physical finding is the hypospadias. Urinalysis is negative for infection. Which of the following is the most appropriate next step? A. Measuring serum creatinine level B. Schedule a renal ultrasound C. Obtain an IV pyelogram D. Cystography E. Performing a circumcision

B. Schedule a renal ultrasound Children who have hypospadias are prone to urinary tract infections and other urinary tract anomalies. They require careful evaluation. A renal ultrasound is a safe way of diagnosing neonatal urinary tract pathology.

A 74-year-old man complains of lower abdominal pain that started today. He has become aware of a periumbilical mass, which occurred concurrently with the development of this mass. His past medical history is significant for hypertension and type 2 diabetes. He had a history of diverticulitis 1 year ago. On physical examination, he is afebrile and appears mildly uncomfortable. On abdominal examination, there is a palpable mass extending from the pubic ramus to the umbilicus with a smooth contour. Laboratory results reveal a BUN of 14 mg/dL and a creatinine of 1.8 mg/dL. Which of the following is the most appropriate next step in the management of this patient? A. Digital rectal examination B. Urethral catheterization C. Abdominal sonogram D. Abdominal and pelvic CT scan E. Percutaneous nephrostomy

B. Urethral catheterization This elderly man has an acute onset of a "lower abdominal mass" and discomfort. Its location and contour suggest that it may be a distended bladder. The modest increase in BUN and creatinine is probably secondary to the impairment to urine production produced by bladder obstruction, with resulting back pressure on the kidney tubules. This probably occurred as a result of benign prostatic hyperplasia. The first order of business would be a urethral catheterization, which would probably allow the disappearance of the "abdominal mass," which is in fact his distended bladder. This should be performed before any imaging such as sonogram (choice C) or CT scan (choice D).

A 20-year-old college student comes to the emergency department because of a sudden onset of excruciating left-sided flank pain radiating to the labia. She states that the flank pain is associated with tenesmus, rectal pain, nausea, and vomiting. She had 2 episodes of gross hematuria. She has no chronic medical conditions and has never had any similar episodes in the past. She has recently started taking "megadoses" of vitamin A, vitamin C, vitamin E, pyridoxine, and methylphenidate (her roommate's) to "help with final examinations." Her temperature is 37.8° C (100° F), blood pressure 130/80 mm Hg, pulse 80/min, and respirations 26/min. Examination shows severe left costovertebral angle tenderness. Radiograph of the kidneys, ureter, and bladder (KUB) shows a radio-opacity in the left ureter. Which of the following is the most likely explanation for these findings? A. Vitamin A excess B. Vitamin C excess C. Vitamin E excess D. Methylphenidate E. Pyridoxine excess

B. Vitamin C excess This patient has a kidney stone, most likely caused by excess supplementation with vitamin C. She has renal colic (flank pain radiating to the groin), gross hematuria, and radiographic evidence of a stone. Some individuals believe that vitamin C can prevent or shorten the duration of a cold; however, studies have failed to demonstrate any differences in occurrence, duration, or severity of a cold with vitamin C administration. Excess vitamin C supplementation can lead to hyperoxaluria and the subsequent development of calcium oxalate stones. Vitamin C is a water soluble vitamin, so increased intake of vitamin C will not accumulate in the body but will still have a physiologic effect on reduced elimination of oxalate in the GI tract.

A patient with essential hypertension is starting diuretic therapy. He has a history of calcium oxalate renal stones. Which of the following diuretics would be most appropriate for this patient? A. Acetazolamide (Diamox) B. Furosemide (Lasix) C. Hydrochlorothiazide D. Spironolactone (Aldactone) E. Triamterene (Dyrenium)

C. Hydrochlorothiazide A thiazide diuretic would be the drug of choice for this patient because it is the only class of diuretics that decreases urinary secretion of calcium. Thiazide diuretics, like hydrochlorothiazide, inhibit the Na+/Cl- cotransporter in the distal convoluted tubule and promote the reabsorption of calcium.

A group of undocumented people is smuggled across the border in a closed metal truck in the middle of summer. When apprised by radio that the border patrol is on their trail, the smugglers abandon the truck, leaving little water for the people to drink. The victims are found and rescued 5 days later. One of them is brought to the emergency department who weighs 80 kg and is awake and alert, with obvious clinical signs of severe dehydration and a serum sodium concentration 155 mEq/L. Which of the following would be the best choice and rate of IV fluid administration? A. 5 L of 5% dextrose in water (D5W) over 2-3 days B. 5 L of D5W over 5-10 hours C. 5 L of 5% dextrose in half normal saline (D5 1/2 NS) over 5-10 hours D. 10 L of D5 1/2 NS over 5-10 hours E. 10 L of normal saline over 2-3 days

C. 5 L of 5% dextrose in half normal saline (D5 1/2 NS) over 5-10 hours Follow the following rough guideline to quantify water loss: Every 3 mEq/L that the serum sodium concentration is above normal represents about 1 L of water deficit With a value of 155, we can assume a water deficit of about 5 L. There is no advantage to the patient in remaining severely volume contracted for several days, thus the replacement should aim for correction in a matter of 5-10 hours rather than 2 or 3 days. Because his losses were incurred slowly, however (over 5 days), his brain has had a chance to adapt to the tonicity change (he is indeed awake and alert). The tonicity correction thus should not happen with the same speed with which the volume is going to be corrected. That delay is achieved by choosing a fluid that is not pure water, but one that has some sodium in it to dampen the effect on tonicity. Half normal saline is a good choice.

A 45-year-old man comes to the emergency department with a 3-day history of hemoptysis. He has had a cough and has also been complaining of sinus congestion and drainage. Furthermore, he has noted blood in his urine. He has no other medical problems and takes no medications. A lung biopsy specimen shows necrotizing inflammation with alveolitis. Urine sediment shows red blood cell casts and dysmorphic red blood cells. Polyangiitis with granulomatosis (formerly called Wegener granulomatosis) is suspected, and steroids are started. The presence of which of the following entities will confirm the diagnosis? A. Elevated angiotensin-converting enzyme (ACE) B. Antiglomerular basement membrane antibodies (anti-GBM) C. Antineutrophilic cytoplasmic antibodies (ANCA) D. Eosinophilia E. X-bodies

C. Antineutrophilic cytoplasmic antibodies (ANCA) Polyangiitis with granulomatosis(formerly known as Wegener granulomatosis) is identified by the classic clinical triad of upper and lower respiratory involvement and renal failure in the form of glomerulonephritis, supported by a positive antineutrophilic cytoplasmic antibody (ANCA) test. The pathologic lesion is an angiitis of small vessels with characteristic tissue necrosis surrounded by mononuclear inflammatory cells, forming noncaseating granulomas. In the lung, this process results in excavation and destruction of the lung parenchyma. The renal lesion is a focal glomerulonephritis that can progress to renal failure.

A 2-year-old girl is brought to the emergency department because she looked "unwell" and has not wanted to get out of bed for 24 hours. She had a diarrheal illness 1 week ago marked by fever and bloody diarrhea, from which she had recovered. Two days ago, however, she developed reduced appetite and lethargy. She has had only 2 wet diapers in the last 36 hours. She was previously a well child and at the regular clinic visits normal growth and development were noted. Examination shows an irritable, pale child who has petechiae and edema. Vital signs are blood pressure 102/74 mm Hg, pulse 136/min, and respirations 26/min. Lung and abdominal examinations are normal. Laboratory studies show: Hemoglobin 7.1 g/dL WBC 22 x 10^9 /L Differential: Polymorphs 38%, Lymphocytes 52%, Monocytes 8% Platelets 80 x 10^9 /L Blood film: Fragmented red blood cells Prothrombin time 12.1 sec Partial thromboplastin time 32 sec Serum sodium 132 mEq/L Serum potassium 6.3 mEq/L Serum chloride 106 mEq/L Serum bicarbonate 16 mEq/L Blood urea nitrogen 32 mg/dL Serum creatinine 3.1 mg/dL Which of the following is the appropriate next step in management? A. Administer a normal saline bolus of 20 mL/kg B. Administer methylprednisolone C. Begin dialysis D. Begin plasmapheresis E. Transfuse red blood cells

C. Begin dialysis This patient has presented in acute renal failure, which is manifested by oliguria (<400-500 mL/24h) elevated blood urea nitrogen and creatinine, and hyperkalemia. Given the clinical picture, the most likely etiology is hemolytic uremic syndrome (HUS). HUS is the most common cause of acute renal failure in young children and most frequently follows an episode of gastroenteritis caused by an enterohemorrhagic strain of Escherichia coli (0157:H7). Endothelial injury to the kidneys leads to renal failure, whereas damage to red blood cells and platelets in the kidneys leads to the thrombocytopenia and microangiopathic hemolytic anemia. The management is supportive care of the renal failure, which is self-limited in most cases. Given the marked elevation of creatinine as well as hyperkalemia in this patient, the most appropriate choice would be to initiate dialysis.

A 67-year-old woman comes to her health care provider because of pain with urination and frequent urination. She has hypertension for which she takes a beta-blocker, but no other medical problems. She states that she is not sexually active. She does not smoke and drinks cranberry juice daily. Examination shows mild suprapubic tenderness and genital atrophy but is otherwise unremarkable. Urinalysis shows 50-100 leukocytes/hpf and 5-10 erythrocytes/hpf. Which of the following is the most likely cause of infection? A. Cardiac disease B. Cranberry juice ingestion C. Hypoestrogenism D. Nephrolithiasis E. Sexual intercourse

C. Hypoestrogenism This patient has a presentation that is most consistent with urinary tract infection (UTI). Two of the major risk factors for uncomplicated UTI are sexual intercourse and hypoestrogenism. Sexual intercourse is believed to lead to urinary tract infection by introducing colonizing bacteria into the bladder. Sexual intercourse has been shown to increase the number of bacteria in the urine up to 10x. Hypoestrogenism is believed to be a risk factor for UTI because it is known that postmenopausal women not receiving estrogen replacement therapy (ERT) are at greater risk for developing a UTI compared with those women who do use ERT. Furthermore, estrogen administration has been shown to prevent recurrent infection. The menopausal woman who has atrophic vaginitis has less protection against UTIs and is at higher risk for the development of these infections.

A 27-year-old Chinese man comes to the clinic because of gross hematuria. The patient has a long history of recurrent hematuria, made worse by upper respiratory infections. He denies arthralgias, fever, chills, or skin rash. There is no family history of kidney disease or hematuria. His temperature is 37° C (98.6° F) and his blood pressure is 120/70 mm Hg. His throat and lungs are clear; his abdomen is soft and nontender. There is no edema, rash, or arthritis. His blood urea nitrogen level is 10 mg/dL and creatinine 0.8 mg/dL. Urine specimen shows 15-20 dysmorphic red blood cells/hpf, and there is +2 protein and 1 red blood cell cast. Which of the following is the most likely diagnosis? A. Alport syndrome B. Goodpasture syndrome C. IgA nephropathy D. Nil disease E. Polyangiitis with granulomatosis (formerly called Wegener granulomatosis)

C. IgA nephropathy The patient most likely has IgA nephropathy. This is the most common idiopathic nephropathy in the world. It will usually present with hematuria following an upper respiratory infection. Proteinuria is frequent, but only rarely is severe. IgA nephropathy is much more common in Asia and South America. With this patient having dysmorphic red blood cells seen on urinalysis, this patient probably has the more aggressive form of IgA nephropathy, that of crescentic nephropathy.

A 54-year-old obese man comes to the health care provider for a routine physical examination. He was diagnosed with type 2 diabetes a year ago. He has been moderately compliant with dietary precautions, and his morning glucose has been persistently 150-200 mg/dL. He is therefore started on glipizide (Glucotrol). One month later metformin (Glucophage) is added because of continued poor control. His other medications are propranolol (Inderal) and nifedipine (Procardia) for hypertension, and naproxen, which he began approximately 2 weeks ago for severe knee pain caused by osteoarthritis. On physical examination his blood pressure is 154/92 mm Hg, and he has a soft fourth heart sound. The remainder of the physical examination is normal. Routine electrolytes reveal BUN 29 mg/dL and creatinine 1.8 mg/dL; both had been normal last year. Which of his medications is most likely responsible for the increase in BUN and creatinine? A. Glipizide (Glucotrol) B. Metformin (Glucophage) C. Naproxen D. Nifedipine (Procardia) E. Propranolol (Inderal)

C. Naproxen NSAIDs such as naproxen may cause a usually mild renal failure, possibly related to a mild interstitial nephritis or vasomotor nephropathy (impaired auto-regulation of GFR caused by inhibition of prostaglandin-dependent renal afferent arteriole vasodilation). Risk for NSAID-induced renal damage is increased in the elderly and in patients who have underlying renal disease.

A 23-year-old African American man with sickle cell disease comes to the emergency department with polyuria and nocturia. He denies visual changes, polyphagia, or weight loss. He also denies orthopnea, paroxysmal nocturnal dyspnea, or dysuria. His temperature is 37° C (98.6° F), blood pressure 105/40 mm Hg, pulse 94/min and regular, and respirations 15/min. Physical examination, including a retinal examination, is unremarkable. There is no lower extremity edema present. Laboratory studies show: Sodium 145 Potassium 5.4 Chloride 100 Bicarbonate 19 Glucose 106 BUN 25 Creatinine 1.5 Urine test shows: Osmolality: 400 mmol pH: normal Protein: 1+ Blood: 1+ LE: negative Nitrite: negative No casts are seen 25 RBCs per hpf 24-hour urine protein: 1.03 g No WBCs or bacteria is seen. A biopsy specimen of the kidney is most likely to show which of the following findings? A. Diffuse red material in the glomeruli and surrounding structures when stained with Congo red B. Increased thickness of the basement membrane with Kimmelstiel-Wilson nodules C. Necrosis of the papilla of the kidney with preservation of the outline of the tubules and collecting ducts D. Widespread effacement of epithelial foot processes seen with electron microscopy with a normal appearance using light microscopy E. Widespread glomerular basement membrane thickening with proliferation of the mesangial and endothelial cells

C. Necrosis of the papilla of the kidney with preservation of the outline of the tubules and collecting ducts This patient has renal papillary necrosis. This may be the only abnormality that is seen in the setting of sickle cell trait. This is a condition that is almost universal in patients who have sickle cell disease. It is characterized by microscopic or gross hematuria, polyuria, and nocturia, and it occurs because infarcts in the renal medulla have reduced the kidney's concentrating ability. Patients often have no symptoms from this disorder. The disorder is often diagnosed when being worked up for unrelated issues. Progression to end-stage renal disease can occur, but is not common.

In clinical trials, an experimental drug is found to cause erectile dysfunction in a large percentage of male patients. Inhibition of which of the following could be responsible for this side effect? A. Conversion of DHT to testosterone B. Forward motility factor C. Nitric oxide synthase (NOS) D. Oxytocin E. Prostaglandins

C. Nitric oxide synthase (NOS) Penile erection is mediated by the parasympathetic nervous system. The neurons involved are termed nonadrenergic, noncholinergic (NANC) autonomic neurons, and they release nitric oxide (NO). NO binds to the iron in the heme molecule of guanylate cyclase, activating it to form cGMP. This results in a decrease in intracellular calcium and subsequent smooth muscle relaxation and vasodilation in the corpus cavernosa, producing an erection. Nitric oxide synthase (NOS) is the enzyme required for the formation of NO from circulating arginine, and androgens are necessary to maintain normal amounts of this enzyme. Inhibition of this enzyme could result in impotence (although most currently used drugs that have impotence as a side effect do not affect NOS).

A 59-year-old African American man has a hard, discrete, 1.5-cm nodule that is felt in his prostate during a routine physical examination. He is completely asymptomatic, and his prostate specific antigen (PSA) 3 months ago was normal for his age. His last rectal examination was performed a year earlier and was unremarkable. He denies any family history of prostate cancer. Which of the following will best establish the diagnosis? A. Clinical follow-up during the ensuing year B. Repeat determination of PSA C. Transrectal needle biopsy of the mass D. Transrectal ultrasound of the prostate E. Transurethral resection of the prostate (TURP)

C. Transrectal needle biopsy of the mass Cancer of the prostate will be discovered early by either the discovery of a hard nodule (as in this case), or the identification of elevated PSA. These are complementary examinations. One may be normal, whereas the other may be revealing. In this case, the recent normal PSA does not exclude the need to biopsy this mass. Although recent studies question the utility of performing a rectal examination in order to assess for prostate cancer, a hard mass, when found, cannot be ignored.

A 5-year-old boy is brought to the clinic because his parents have noticed swelling around his eyes for the past 3 days. The child was previously well and was last seen for an annual well-child visit 1 month ago when his weight was 18.5 kg (40.8 lb). He has been urinating less than usual but he is otherwise well. No change in the urine color was noticed and there was no fever, diarrhea, or vomiting. Examination reveals an alert and active child. Blood pressure is 98/66 mm Hg. There is periorbital edema as well as pitting edema of the ankles. Lung and abdominal examinations are normal and there is no skin rash. Laboratory studies show: CBC: Hemoglobin 12.4 WBC 8.4x10^9 Differential: Polymorphs 48%, lymphocytes 42%, monocytes 8% Platelets 280x10^9 Chemistry: Serum sodium 138 Serum potassium 4.3 Serum chloride 106 Serum bicarbonate 26 Serum calcium 7.1 BUN 8 Serum creatinine 0.6 Serum triglycerides 580 Serum albumin 2.1 Urine: pH 6.5 Specific gravity 1.015 Glucose negative Protein large Blood negative Leukocyte esterase negative Which of the following is the most appropriate next step in management? A. Albumin infusion B. Cyclophosphamide (Cytoxan) C. Furosemide (Lasix) D. High-protein diet E. Prednisone

E. Prednisone This patient has nephrotic syndrome, based on the presence of edema, hypoalbuminemia, hyperlipidemia, and proteinuria >3.5 g. Nephrotic syndrome in children is most often idiopathic and common between ages 1 and 8. Most cases are caused by minimal change disease (Nil disease) in which the renal biopsy specimen reveals few or no changes on light microscopy, but under electron microscopy there is effacement of the renal tubular epithelial foot processes. The urine shows heavy protein (>3.5 g) and no abnormal urinary sediment or hematuria. Children ages 1-8 are likely to have steroid-responsive nephrotic syndrome, and thus initial therapy with prednisone is the most appropriate choice. Parents should also be advised to restrict salt intake until the edema subsides.

A 57-year-old man comes to the healthcare provider because of "erectile problems." He says that he has not been able to have an erection in months. He has seen so many television commercials lately that he expects a cure in the form of a "little blue pill." He is married, has 3 children, works as a trader on the stock exchange, and is an avid cyclist. He takes no medications, drinks no alcohol, and has no serious medical conditions. Physical examination is unremarkable. Which of the following questions is most likely to help determine the cause of this patient's condition? A. "How often do you ride your bicycle?" B. "Are you sexually attracted to your wife?" C. "Are you feeling unusually anxious lately?" D. "Do you have nocturnal or early morning erections?" E. "Do you love your wife?"

D. "Do you have nocturnal or early morning erections?" The main issue in evaluating impotence is distinguishing between functional erectile dysfunction causes and organic causes. The presence of nocturnal or early morning erections basically eliminates the organic causes and leads to a diagnosis of functional erectile dysfunction. The most common psychologic causes include anxiety and depression. Nocturnal penile tumescence, which occurs during REM sleep, can be assessed in a sleep laboratory. If nocturnal erections are not present, the impotence is most likely due to an organic cause. The most likely causes are testicular failure, hyperprolactinemia, medications, alcohol, opioids, nicotine, trauma, priapism, diabetes, vascular disease, and neurologic diseases, such as diseases of the spinal cord and loss of sensory input. Physical examination should include a detailed genital examination, evaluation for signs of feminization, and neurologic and vascular examinations.

A man with prostatic hyperplasia is placed on finasteride (Proscar). To which of the following classes of medications does this agent belong? A. Alpha-blockers B. Gonadotropin-releasing hormone inhibitors C. Direct testosterone antagonists D. 5-Alpha reductase inhibitors E. Phosphodiesterase inhibitors

D. 5-Alpha reductase inhibitors Finasteride and dutasteride are 5-alpha reductase inhibitors that block the conversion of testosterone to dihydrotestosterone, which results in the decrease of prostate size by about 25%. These agents are used to cause symptomatic improvement in patients with urinary outlet obstruction due to prostatic hyperplasia.

A 62-year-old man reports an episode of gross, painless hematuria. There is no history of trauma, and further questioning determines that he had hematuria lasting throughout the voiding process, rather than initial or terminal hematuria. The man does not smoke and has had no other symptoms referable to the urinary tract. He has no known allergies. Physical examination, including rectal examination, is unremarkable. His serum creatinine level is 0.8 mg/dL, and except for the presence of many red blood cells, urinalysis is normal and shows no red blood cell casts. Hematocrit is 46%. Which of the following is the most appropriate initial step in the workup? A. Coagulation studies and urinary cultures B. Place a Foley catheter C. PSA determination and prostatic biopsies D. CT scan with IV contrast of both kidneys, ureters, and bladder, and cystoscopy E. Retrograde cystogram and pyelograms

D. CT scan with IV contrast of both kidneys, ureters, and bladder, and cystoscopy Although most patients with hematuria have benign disease, silent hematuria can be caused by renal, ureteral, or bladder cancer, and so those malignant processes must be effectively ruled out. An IV contrast-enhanced CT scan has now largely replaced IV pyelograms to assess for anatomic and functional abnormalities of the kidneys and collecting system. It gives a much better picture and greater detail about the anatomy and consistency of lesions found in the kidney parenchyma and ureters. Direct visualization of the bladder mucosa by cystoscopy is the only way to rule out bladder cancer. If filling defects are identified via the CT scan, cystoscopy would be indicated.

A 45-year-old woman reports that she has been having increased involuntary loss of urine after experiencing the sudden need to urinate. This is especially pronounced when she is feeling nervous or while sitting at her desk. Jogging does not worsen the incontinence. She has not had these symptoms in the past and is otherwise healthy. She is not taking any medications and has never been pregnant. On physical examination she is afebrile with stable vital signs. Her abdomen is benign, and vaginal examination reveals no prolapse. Sensation in all extremities is intact, with good motor strength. Her gait is normal, and reflexes are intact. Her workup reveals a negative urinalysis, and blood cultures and urine cultures are all negative. Serum glucose level and glycosylated hemoglobin level is normal. Which of the following tests would be the most appropriate next step in diagnosis? A. IV pyelogram (IVP) B. Stress testing C. Q-tip test D. Cystometry E. Cystoscopy

D. Cystometry The patient most likely has detrusor instability or dyssynergia. This patient most likely has urge incontinence since she has difficulty delaying urination once the urge is present. She does not seem to have a neurologic, infectious, or systemic (e.g., diabetes) disorder. Because she can tolerate jogging without a problem, she probably does not have stress-induced incontinence (SUI). The diagnosis thus will probably be made by cystometry to assess her urodynamics. This test is also used to evaluate bladder capacity and tone.

A 29-year-old woman comes to the health care provider because of fevers and back pain. She is otherwise healthy with no significant past medical history. Examination is significant for a temperature of 38.3º C (101º F), moderate costovertebral angle tenderness, leukocytosis, and white blood cells and red blood cells in the urine. The patient is diagnosed with pyelonephritis and admitted to the hospital for therapy with intravenous antibiotics. Over the next two days, she rapidly improves, and by hospital day 3, she is tolerating oral intake, voiding without difficulty, feeling no pain, and she has not had a fever for 48 hours. Which of the following is the most appropriate next step in management? A. Continue intravenous antibiotics for 2 weeks B. Discharge home and recommend post-coital prophylaxis C. Discharge home off all antibiotics D. Discharge home to complete a 2-week course of oral antibiotics E. Obtain surgical evaluation

D. Discharge home to complete a 2-week course of oral antibiotics This patient has had an uncomplicated course of pyelonephritis thus far. Pyelonephritis is an infection of the kidney. Patients with pyelonephritis typically present with some combination of back pain, dysuria, hematuria, frequency, urgency, fevers, chills, nausea, and vomiting. Examination often shows an elevated temperature, costovertebral angle tenderness, leukocytosis, and white cells and red cells in the urine. Completely uncomplicated cases of pyelonephritis can be treated on an outpatient basis. When there are any complicating factors (e.g., concern for sepsis, pregnancy, old age, or other medical illnesses), the patient should be admitted to the hospital for intravenous antibiotics. However, once the patient's condition has improved and she is tolerating oral intake, she may be discharged home to complete a 2-week course of antibiotics. When discharged, however, she should be given strict instructions and precautions regarding the need to return for recurrence of the symptoms or worsening condition.

A 78-year-old retired psychiatrist goes to his primary healthcare provider because of difficulty urinating. He constantly feels the need to urinate and wakes up many times in the middle of the night to use the bathroom. Unfortunately. his urine stream is very weak and he always feels like there is "more left" when he is done. He is otherwise very healthy, plays tennis daily, and eats a healthy diet. He takes no medications, drinks a glass of wine with dinner, and does not smoke. His temperature is 36.7°C (98°F). Physical examination reveals a firm, nodular prostate. The remainder of the examination is unremarkable. Laboratory studies show a prostate-specific antigen level 10 ng/mL. Urinalysis is unremarkable. A biopsy specimen of the prostate gland reveals adenocarcinoma with a Gleason score 4. When the patient arrives for his follow-up visit to receive his diagnosis, he says that he decided that he doesn't want to hear what the biopsy showed and that he would rather continue on with his life the way that it is. The health care provider tries to determine why he does not want to hear about it, and the patient replies that he decided that it is not necessary to worry about things that are out of his control. He wouldn't undergo any treatment, anyway, if he has cancer. A psychiatrist finds him capable to make his own decisions. After once again offering to provide the information and with the patient again refusing to hear it, which of the following is the most appropriate next step in management? A. Call the patient's wife and tell her the diagnosis B. Have the entire family come to the office with the patient to tell all of them the diagnosis C. Have the patient see another health care provider and have the other health care provider advise him to hear about his diagnosis and prognosis D. Honor the patient's wishes E. Tell the patient his diagnosis

D. Honor the patient's wishes Patients have the right to decline to hear their diagnosis and prognosis. As long as the health care provider tries to offer the information and discusses the reasons for the patient not wanting to hear about it, the patients should not be forced to hear the information against their will (choice E).

A 27-year-old woman comes to the health care provider complaining of fever and back pain. She states that a few days ago she had burning with urination. Over the next few days she developed fever with chills and had pain on the right side of her back. She is so sick that she cannot tolerate oral intake. Previous to this acute incident, she had no medical problems and she was taking no medications. Her temperature is 38.9°C (102°F), blood pressure 110/70 mm Hg, pulse 102/min, and respirations 16/min. Examination shows a patient in mild distress with shaking chills and right costovertebral angle tenderness. Leukocyte count is 18,000/mm3. Urinalysis shows 100 leukocytes/high-power field. Which of the following is the most appropriate next step in management? A. Observation only B. Spinal MRI scan C. Outpatient management with oral trimethoprim-sulfamethoxazole D. Hospital admission and initiation of IV ciprofloxacin (Cipro) E. Hospital admission and administration of a 2-week course of IV tetracycline

D. Hospital admission and initiation of IV ciprofloxacin (Cipro) This presentation is consistent with pyelonephritis: some combination of back pain, fevers, chills, dysuria, nausea, and vomiting. Examination often shows an elevated temperature, costovertebral angle tenderness, and elevated leukocyte count. Urinalysis may demonstrate positive nitrite and leukocyte esterase testing. Urine sediment often reveals WBC, RBC, and white blood cell casts. Pyelonephritis can be managed on an outpatient basis if the patient is otherwise healthy, has no complicating factors, and is reliable to return if the condition worsens. It cannot be managed on an outpatient basis if there is any evidence of sepsis or if the patient is unable to tolerate oral intake. This patient, with high fever, shaking chills, and elevated leukocyte count, may have sepsis and should therefore be admitted to the hospital for IV antibiotics. She is also unable to tolerate oral intake. Treatment is IV ciprofloxacin, ceftriaxone, gentamicin, ampicillin/sulbactam, or ofloxacin. Once the patient is afebrile, her condition is improving, and she is able to tolerate oral intake, she may be converted to an oral antibiotic regimen to complete at least a 7-day course depending upon the treatment that was chosen.

A 3-week-old African American boy is brought to the emergency department because of a generalized seizure 2 hours ago. The infant is highly irritable with incessant high-pitched crying. The infant's weight is 2.5 kg (250 g below birth weight), blood pressure is 70/40 mm Hg, pulse 145/min, and respirations 50/min. Laboratory results show: Blood glucose 120 mg/dL Urea nitrogen 50 mg/dL Serum sodium 170 mEq/L Serum calcium 8.5 mg/dL Serum magnesium 1.5 mg/dL Which of the following is the most likely cause of this infant's seizure? A. Hypocalcemia B. Hypoglycemia C. Hypomagnesemia D. Hypernatremia E. Meningitis

D. Hypernatremia The level of serum sodium in this patient is 170 mEq/L. Infants who have hypernatremic dehydration are irritable and lethargic and have a high-pitched cry. This type of dehydration results from a greater loss of hypotonic fluid than sodium and accounts for about 15% of cases of dehydration. Because the patient has no history of diarrhea or vomiting, the hypernatremia may be caused by inadequate supply of mother's milk that does not match the insensible water loss. Another cause can be the high concentration of sodium in mother's milk. Generally, after the child's birth, sodium in the colostrum decreases from its highest level to its lowest level by week 4. Some mothers continue to excrete high sodium in their milk, however, and can potentially cause recurrent hypernatremia and in some cases, intracranial hemorrhage in the infant.

A 62-year-old man is brought to the emergency department because of status epilepticus. He has a history of small cell lung cancer. He has no history of hypertension, diabetes, thyroid disease, or congestive heart failure. Examination reveals a male in a postictal state, with a blood pressure of 130/90 mm Hg. A funduscopic examination reveals no papilledema, and his neck is supple. Lungs are clear, and his heart examination is normal without jugular venous distention. There is no edema or clubbing. Laboratory studies show: Serum: Sodium 112 mEq/L (normal, 135-147 mEq/L) Potassium 4.0 mEq/L (normal, 3.5-5.3 mEq/L) Urea nitrogen 10 mg/dL (normal, 8-18 mg/dL) Creatinine 0.8 mg/dL (normal, 0.8-1.4 mg/dL in males) CT scan of the head is normal. Chest radiograph reveals a right perihilar mass unchanged from a film 1 month ago. Which of the following is the most appropriate initial step in management? A. Demeclocycline (Declomycin) B. Fluid restriction C. Hydrocortisone D. Hypertonic saline E. Thyroxine

D. Hypertonic saline Hypertonic saline would be the most advisable treatment. The patient has hyponatremia and seizures, which would suggest an acute decrease in sodium. There is no mention of a brain lesion to explain the seizures. The patient most likely has a syndrome of inappropriate anti-diuretic hormone secretion (SIADH) caused by his lung cancer. Furosemide may be given with the hypertonic saline to promote water loss.

A 7-year-old boy presents to the health care provider because his mother noticed a smoky color to his urine. On questioning the mother, it is revealed that the child had a sore throat several weeks ago that was left untreated. Physical examination reveals hypertension and mild generalized edema. Urinalysis is significant for red blood cell casts. Which of the following accurately describes the microorganism responsible for this child's illness? A. It causes alpha-hemolysis on blood agar. B. It is catalase-positive. C. It is coagulase-positive. D. It is sensitive to penicillin. E. It is sensitive to metronidazole.

D. It is sensitive to penicillin. This is one of the higher-order questions the exam favors. In this case, you need to figure out which disease the child has, which organism causes the disease, and which of the listed features is true of the microorganism. The disease in question is poststreptococcal glomerulonephritis, as evidenced by the smoky urine, hypertension, edema, and red blood cell casts in the urine sediment. The history of the prior sore throat is a tip-off that this is a nonsuppurative sequela of an infection caused by Streptococcus pyogenes (Group A β-hemolytic streptococci). S. pyogenes can be differentiated from Streptococcus pneumoniae and viridans streptococci by its hemolytic pattern; it is beta-hemolytic, whereas the others are alpha-hemolytic (choice A). This organism is sensitive to penicillin. Other important things to remember about S. pyogenes are its many virulence factors, including M protein, antiphagocytic capsule, hyaluronidase, streptolysins O and S, and erythrogenic toxins. Patients with poststreptococcal glomerulonephritis have a nephrogenic strain for which treatment with penicillin will not prevent since this glomerulonephritis results from the immune complexes that are formed as a result of this infection.

A 27-year-old-man with no significant past medical history comes to the health care provider for an infertility evaluation. He and his wife have been trying to conceive for 2 years without success. A complete infertility workup of his wife found no abnormalities. History reveals that the patient had normal childhood development but late puberty. He had problems throughout his life in school secondary to what counselors told him was a moderate learning disability. Physical examination reveals a tall man with long legs and arms that seem out of proportion to the rest of the body. His testes are small, and moderate gynecomastia is present. Which of the following findings is consistent with this patient's condition? A. Decreased luteinizing hormone, follicle-stimulating hormone, and testosterone B. Elevated prolactin level C. Increased luteinizing hormone to follicle-stimulating hormone level D. Karyotype 47,XXY E. 21-Hydoxylase deficiency

D. Karyotype 47,XXY This patient has Klinefelter syndrome. It is best diagnosed with a 47,XXY karyotype, though variants with 46,XY exist. The condition is characterized by small testes, disproportionately long arms and legs (secondary to delayed epiphyseal plate closure in puberty), infertility, and gynecomastia. Mental retardation and learning disability are common. Many men are not diagnosed until adulthood when they seek medical attention for infertility. Our patient clearly has these traits.

A 3-year-old child develops severe generalized edema following a viral infection. On the basis of clinical chemistry tests, a renal biopsy is performed, with normal light microscopic findings. Which of the following abnormal laboratory values might be expected in this individual? A. Decreased alpha-2 globulin levels B. Decreased fibrinogen C. Increased serum calcium levels D. Low serum albumin levels E. Red blood cell casts in the urine

D. Low serum albumin levels This child has minimal change disease, which is the major cause (more than 90% of cases) of nephrotic syndrome in children aged 2 to 6 years. The most prominent clinical chemistry finding in these patients is massive proteinuria. The urinary protein in minimal change disease, in contrast to other causes of nephrotic syndrome, is often composed predominantly of albumin. Many other clinical chemistry changes may also be seen, including decreased serum albumin levels, hyperlipidemia, increased serum levels of alpha-2- and beta-globulins, decreased IgG, and increased fibrinogen.

A 24-year-old woman comes to the health care provider because of burning with urination. She states that every time she urinates there is pain and that she has a feeling that she constantly needs to urinate even though only a little comes out. She has never had any similar symptoms before. She has no medical problems and no known drug allergies. Examination is unremarkable. Urinalysis demonstrates that the urine is positive for leukocyte esterase and nitrites. Which of the following is the most appropriate pharmacotherapy? A. Intramuscular ceftriaxone B. IV levofloxacin C. Oral levofloxacin for 7 days D. Oral trimethoprim-sulfamethoxazole for 3 days E. Wait for the culture results to institute therapy

D. Oral trimethoprim-sulfamethoxazole for 3 days This patient likely has an uncomplicated urinary tract infection (UTI). Patients with UTI often present with dysuria, frequency, and urgency. Physical examination is often unremarkable, although there may be some suprapubic tenderness if cystitis is the predominant infection rather than urethritis. Urine "dip" will often be positive for leukocyte esterase and nitrites. Microscopic urinalysis will often show the presence of white blood cells and red blood cells. Escherichia coli is the offending organism in about 80% of cases, with Staphylococcus saprophyticus being the next most likely causative organism. Treatment of an uncomplicated UTI is a 3-day course of oral antibiotics. Trimethoprim-sulfamethoxazole has been shown to be safe, effective, and cost-effective in the treatment of uncomplicated UTIs.

A healthy 14-year-old boy is brought to his health care provider for a routine health maintenance visit. A urinalysis reveals 1+ proteinuria. There is no hematuria or bacteriuria. His physical examination is unremarkable, revealing no edema and a normal blood pressure. There is no family history of renal disease. Which of the following is the most likely diagnosis? A. Acute glomerulonephritis B. IgA nephropathy C. Minimal change disease D. Orthostatic proteinuria E. Urinary tract infection

D. Orthostatic proteinuria Orthostatic proteinuria is very common. It occurs when the total urinary protein excretion rate is higher while the child is in an upright position. It is asymptomatic, and there is no associated increase in renal disease. The diagnosis is made by obtaining a split 24-hour urine collection. Half is collected while in the recumbent position and half while standing.

A 28-year-old man is 7 hours status-post an appendectomy and complains of severe lower abdominal-suprapubic pain. On examination his incision is clean, dry, and intact and without pain to the touch. Palpation of the suprapubic area produces intense discomfort. He has not urinated since the procedure. The nurse notes that he is tachycardic with a regular pulse of 110/min. Which of the following is the most appropriate next step in management? A. Administer a beta-blocker such as metoprolol B. Administer an anti-adrenergic agent such as doxazosin C. Begin IV antibiotics D. Perform a straight catheterization of the bladder E. Place a nasogastric tube (NGT)

D. Perform a straight catheterization of the bladder Post-surgical patients often have urinary retention as part of their normal, immediate postoperative course; therefore, indwelling catheters are often placed. For a young patient such as this, however, these catheters are not always placed. The reasons for this are variable, but mostly revolve around the expectation of a speedy and uneventful postoperative course and discharge home. In patients who do not have indwelling catheters, urinary retention and associated pain are frequent concerns. Voiding problems postoperatively may be partly caused by the use of anticholinergic medications prior to or during surgery.

A 23-year-old dancer comes to the urgent care clinic with a chief complaint of weakness. She denies any other symptoms, including nausea, vomiting and diarrhea. Her blood pressure is 80/40 mm Hg. There is no edema and the lungs are clear. Laboratory analysis of the serum shows: Sodium 130 mEq/L Potassium 2.2 mEq/L Bicarbonate 29 mEq/L Magnesium 2.0 mg/dL Calcium 9.0 mg/dL The most likely cause of the patient's weakness is an abnormality in which of the following? A. Bicarbonate B. Calcium C. Magnesium D. Potassium E. Sodium

D. Potassium The low potassium is the most likely cause of the weakness. The patient may be a diuretic abuser or may vomit as part of an eating disorder.

A 45-year-old man consults his health care provider because of dysuria. The patient is treated with antibiotics, but symptoms recur 1 week after antibiotic therapy is stopped. A different antibiotic is tried, but symptoms again recur after cessation of the antibiotic. Rectal examination demonstrates an enlarged prostate with areas of tenderness and a fluctuant mass. Which of the following is the most likely diagnosis? A. Benign prostatic hyperplasia B. Chronic nonbacterial prostatitis C. Prostadynia D. Prostatic abscess E. Prostatic carcinoma

D. Prostatic abscess The patient has a prostatic abscess. Typical presentation is age 40-60, and is consequently somewhat younger than the ages at which BPH and prostate cancer become major problems. Infecting organisms include aerobic gram-negative bacilli and Staphylococcus aureus. Prostatic abscess should be suspected when a man develops repeated urinary tract infections that seem to get better with antibiotic therapy, only to recur later. The most important diagnostic clue, if detectable, is the presence of a fluctuant mass in the prostate on rectal examination. Some patients have only prostatic enlargement or even no positive findings on physical examination. Patients may have normal urine, though it is more usual for an organism to be cultured at some point. Prostatic ultrasound may be helpful if the diagnosis is suspected. A few cases are even picked up at the time of prostatic resection for benign prostatic hyperplasia or other disease. Treatment is evacuation of the abscess by a transurethral or perineal route, followed by appropriate antibiotics.

A previously healthy 5-month-old infant is brought to the emergency department because of diarrhea and decreased activities for the past 3 days. He is generally healthy and is up to date on his recommended vaccinations. His father has idiopathic epilepsy and his mother had gestational diabetes but now has normal blood glucose levels. On physical examination, the infant's pulse is 162/min. His anterior fontanelle is sunken, and he has skin tenting. Laboratory investigation reveals: Sodium 165 mEq/L Potassium 5.8 mEq/L Chloride 128 mEq/L Bicarbonate 14 mEq/L Creatinine 0.9 mg/dL Blood urea nitrogen 49 mg/dL Glucose 154 mg/dL Eight hours after IV fluid therapy is initiated, the infant develops a generalized seizure. Which of the following is the most likely etiology of the seizure? A. Hyperglycemia B. Hyperkalemia C. Idiopathic epilepsy D. Rapid correction of hypernatremia E. Rapid correction of metabolic acidosis

D. Rapid correction of hypernatremia The infant in this clinical vignette has developed hypernatremic (Na+ 165 mEq/L) dehydration from the diarrhea. Hypovolemic patients who have hypernatremia have a relatively greater loss of water than of sodium. Initial therapy is administration of normal saline or Ringer's lactate to restore an effective circulating plasma volume. Five percent albumin solution or plasma also can be used. Patients require a hypotonic solution containing salt to restore the Na+ deficit (2-5 mEq/kg of body weight) and to begin the Na+ maintenance (3 mEq/kg of Na+) in a solution containing 20-40 mmol/L of KCl and 5% glucose. For serum Na+ concentration 150-160 mEq/L, this volume should be given over a 24-hour period. Because extracellular fluid osmolarity may fall more rapidly than the brain can dissipate the idiogenic osmoles generated to protect intracellular osmolarity, an elevated serum Na+ concentration should be corrected by maximum 12 mEq/L per day. For serum Na+ concentration >160 mEq/L, the rehydration should be spread out over the number of days necessary to lower the Na+ concentration to 150 mEq/L at a rate of 12 mEq/day (e.g., 2 days for an Na+ of 170 mEq/L). Both the daily fraction of the deficit and the daily maintenance requirement should be provided. The degree of hypotonicity of the fluid administered is less important than the rate of correction. If hypernatremia is corrected too rapidly, the brain cells can be swollen beyond the cell volume restored by the osmoles, resulting in seizures.

A 24-year-old woman sustains multiple injuries in a car accident, including a pelvic fracture. She is hemodynamically stable. Initial assessment shows no vaginal or rectal injuries; however, when a Foley catheter is inserted, bloody urine is recovered. Which of the following would be the best way to evaluate her urologic injury? A. Ultrasound of the bladder B. IV pyelogram C. Cystoscopy D. Retrograde cystogram including post-void films E. Retrograde cystogram including views of the ureters

D. Retrograde cystogram including post-void films Bloody urine plus pelvic fracture equals bladder injury in either gender, or bladder or urethral injury in the male. In this case, with the very short and well-protected female urethra injury not being suspected, only the bladder is the obvious candidate. Injecting dye and taking radiographs will show the extravasation, but it is important to include post-void films, because extravasation at the bladder neck can be obscured by the dye that is filling the bladder.

A 14-year-old boy slides down a banister and crashes into a large ornamental knob at its base, thereby injuring his scrotal contents. He presents in the emergency department with acute testicular pain and a scrotal hematoma the size of a grapefruit. He is able to void normally, and his urine does not contain blood. Rectal examination is unremarkable. Findings from which of the following tests will most likely determine further therapy? A. Aspiration of scrotal contents B. Retrograde cystogram C. Retrograde urethrogram D. Scrotal ultrasound E. Scrotal surgical exploration

D. Scrotal ultrasound The clinical findings do not suggest urethral injury, but testicular trauma is a potential injury that could surgical intervention. Ultrasound would show the extent of the trauma to the testicle and would help to determine the appropriate treatment intervention.

A 53-year-old woman with no past medical history complains of incontinence. She thinks that she must have "a small bladder capacity" because she has to go to the restroom so frequently. She also complains that she tends to lose her urine when she coughs or laughs. Particularly embarrassing is the fact that when she goes to the gym, she has to wear an undergarment while she does light weightlifting so she does not lose her urine in public. A recent urinalysis and urine culture she had at her employee health center were unremarkable. She denies recent back or head injury, neurologic dysfunction, and renal, bladder/urethral/ureteral disorder. She denies any psychological disorder and appears well-adjusted. She only takes oral contraceptive pills regularly; she does not drink or smoke but admits to drinking 3 cups of coffee a day. Which of the following is the most likely diagnosis? A. Drug-induced incontinence B. Functional incontinence C. Overflow incontinence D. Stress incontinence E. Urge incontinence

D. Stress incontinence Stress incontinence is an involuntary loss of bladder function. Often the precipitating events for stress incontinence involve activities that increase intra-abdominal pressure, such as laughing or coughing. Valsalva-type maneuvers, such as weightlifting, are notorious for causing stress incontinence. This condition is seen in women, and its incidence increases with age. It is treated conservatively with pelvic floor exercises (Kegel maneuvers) that strengthen the musculature of the bladder outlet. Other modalities that can be used include medications (alpha-agonists, estrogen), behavioral therapy (biofeedback), and surgery (bladder neck suspension).

A 57-year-old painter comes to his health care provider for evaluation of difficulty voiding for the past few years. He describes a delay when he "attempts to go" and that he often wets his underwear with additional urine after he has finished. He is married and has a teenage child. His past medical history is remarkable for essential hypertension that he claims is diet controlled. Blood pressure is 140/90 mm Hg and pulse 82/min. Physical examination is remarkable for a prominent apical impulse, laterally displaced, and faint abdominal bruit. The prostate is nontender and appears large with no palpable nodules or irregularities. Which of the following is the most appropriate medication to start in this patient? A. Lisinopril (Zestril, Prinivil) B. Metoprolol (Lopressor, Toprol) C. Nifedipine (Procardia) D. Terazosin (Hytrin) E. Verapamil (Calan, Isoptin)

D. Terazosin (Hytrin) An important concept to recognize in the treatment of medical conditions is that certain medications overlap syndromes and are efficacious in many areas. This "co-treatment" option maximizes the benefits of each drug in a regimen and often addresses 2+ issues simultaneously. Terazosin an alpha-1 receptor antagonist, which will lower a borderline elevated blood pressure plus will help the patient to void. Remember that alpha-1 receptors are responsible for the contraction of vascular smooth muscle. Also, this class of drugs is useful in the treatment of benign prostatic hyperplasia (BPH) by relaxing the smooth muscle tissue of the prostate and bladder, improving urine flow and reducing BPH symptoms.

A 7-year-old boy is brought to the clinic for a lifetime history of bed-wetting. He has otherwise been completely healthy and has met all developmental milestones. His parents deny a history of trauma, and the history is not consistent with abuse. The patient has been wetting every night but not during the daytime. He has no incontinence. Which of the following is the most appropriate next step in his evaluation? A. IV pyelogram B. Renal ultrasound C. 24-hour urine collection D. Urinalysis E. CT of pelvis

D. Urinalysis Given the fact that this patient has had a lifelong history of bedwetting, the initial evaluation will include a urinalysis to rule out infection or bleeding. No neurologic dysfunction exists in this case.

A 62-year-old woman had an abdominal hysterectomy and salpingo-oophorectomy 3 days ago. She had an indwelling bladder catheter during the procedure, which was removed in the recovery room. She has been voiding at will since then. She also had compression pneumatic stockings on both lower extremities during the operation. She began ambulation on the first postoperative day, and has been as active as possible under the circumstances, including faithful adherence to a prescribed program of incentive spirometry. On the evening of the third postoperative day she spikes a fever, with a temperature to 39.4ºC (103ºF). Which of the following is the most likely source of the fever? A. Atelectasis B. Deep thrombophlebitis C. Intra-abdominal abscess D. Urinary tract infection E. Wound infection

D. Urinary tract infection The timing is the major clue. Fever on postoperative day 3 usually originates from the urinary tract. The circumstances for a urinary tract infection are also here: this woman had instrumentation of her urinary tract during the surgical procedure.

A 4-year-old boy falls from the jungle gym at preschool. He sustains minor abrasions and contusions, and is taken care of by the school nurse. His parents take him that same afternoon to his regular health care provider and demand "a thorough checkup" for possible internal injuries. The provider complies, and a complete physical examination is normal. His hemoglobin is 14 g/dL, and a urinalysis shows the presence of microscopic hematuria. There are no additional signs of bleeding. Which of the following is the most appropriate next step in management? A. CT scan of the abdomen and pelvis B. Reassure the parents that microscopic hematuria from minor trauma will resolve spontaneously C. Serial hemoglobin and hematocrit determinations D. Urologic workup, starting with an ultrasound E. Retrograde ureterogram and cystogram

D. Urologic workup, starting with an ultrasound Although major urologic injury is unlikely, this history warrants an investigation. In children, microscopic hematuria that follows minor trauma may be an indication that a congenital anomaly is present. In this workup, the first step would be an ultrasound.

A 15-year-old boy presents to the physician assistant with a chief complaint of burning on urination. A purulent urethral discharge is noted on physical examination; a Gram stain of the smear is consistent with gonorrhea. The patient explains that he had sexual intercourse 7 days earlier. He admitted having sex with a consenting female friend who is the same age as the patient. The physician assistant may treat the patient A. but must have both parents' consent B. but must inform the parents C. with one parent's consent D. without a parent's consent or informing the parents E. without informing parents, but must inform the court

D. without a parent's consent or informing the parents No states specifically mandate parental involvement for a minor to obtain contraception, prenatal care and delivery services, diagnosis and treatment of an STI, outpatient alcohol and/or drug treatment, or outpatient mental health services. In these situations, the physician assistant may go ahead and treat; informing the parents is the physician assistant's option, not a requirement. Since the age of the partner was within two years of the partner, this is not considered statutory rape. The other choices conflict with the correct answer.

A 7-year-old girl complains of increased urinary frequency, dysuria, and itching on urination. Her urinalysis is consistent with a urinary tract infection. This is her 20th infection in the past year, despite adequate antibiotic coverage. Further imaging of her bladder, kidneys, and ureters reveals findings that are consistent with vesicoureteral reflux. Which of the following is the next appropriate step? A. CT scan of the pelvis B. IV antibiotics for 2 weeks C. IV pyelogram D. Renal arteriogram E. Antireflux surgery

E. Antireflux surgery Vesicoureteral reflux is a common anatomic cause of recurrent urinary tract infections in children. Patients who have reflux have retrograde flow of urine into the ureters and/or kidney before voiding. This occurs because of an incompetent vesicoureteral valve. Many children outgrow mild degrees of reflux if they are maintained on prophylactic antibiotics. Moderate to severe cases of reflux frequently require surgery. Failure of antibiotics is also an indication for surgery.

A 76-year-old man is brought to the emergency department (ED) by ambulance for altered mental status. The patient has a past medical history significant for multiple myeloma. He was last seen by his health care provider 4 weeks ago, and he was reportedly well at that time. Two days ago the family noticed that the patient had increased confusion, which continued over the next 48 hours; the patient slowly became obtunded, and he could no longer follow simple commands. On physical examination, his blood pressure is 90/50 mm Hg, his pulse is 110/min, his mucous membranes are dry, and there is no jugular venous distention. His lungs are clear and he is alert only to person and cannot follow commands. His deep tendon reflexes are brisk. Serum chemistries and toxicologies are sent as part of the evaluation. Which of the following is the most appropriate initial step in management? A. Administer IV salmon calcitonin B. Administer IV pamidronate C. Administer IV bicarbonate D. Begin aggressive diuresis with furosemide E. Begin aggressive isotonic IV hydration followed by diuresis if there is a lack of improvement

E. Begin aggressive isotonic IV hydration followed by diuresis if there is a lack of improvement This patient most likely has hypercalcemia secondary to his multiple myeloma. The issue here is the acute management of symptomatic hypercalcemia. These patients are very dehydrated from the solute-induced diuresis that also elevates the serum concentrations secondary to contraction of the plasma volume. The first priority of treatment is to expand the extracellular volume with isotonic saline. Once the patient has had adequate volume expansion, loop diuretics may be administered to allow calcium excretion via the urine.

A 55-year-old man with a history of recurrent calcium-containing renal stones presents to the emergency department with excruciating flank pain and blood in the urine. This patient is likely to have which of the following underlying disorders? A. Anemia of chronic disease B. Chronic Proteus infection C. Factor VIII deficiency D. Hyperaldosteronism E. Hyperparathyroidism

E. Hyperparathyroidism This patient is experiencing the very painful passage of a renal stone, which is often accompanied by hematuria. His history of recurrent urolithiasis with calcium-containing stones implies a disorder in the regulation of calcium concentration. Hyperparathyroidism is associated with increased parathormone (PTH) levels, which can produce hypercalcemia, hypercalciuria, and ultimately renal stones.

A 72-year-old man has a 3-mm ureteral stone impacted at the ureterovesical junction. He has been having mild ureteral colicky pain for about 12 hours, and he has been given fluids and analgesics in the expectation that he will spontaneously pass the stone. He then has shaking chills and spikes a temperature of 40° C (104° F). When seen shortly thereafter, he has flank pain and looks quite ill. Which of the following is the most appropriate next step in management? A. Addition of IV antibiotics to the current therapeutic regimen B. Crushing and extraction of the stone via cystoscopy C. Extracorporeal shock wave lithotripsy and parenteral antibiotics D. Immediate insertion of a suprapubic catheter into the bladder E. IV antibiotics and immediate decompression of the urinary tract above the stone

E. IV antibiotics and immediate decompression of the urinary tract above the stone The combination of obstruction and infection in the urinary tract constitutes a dire emergency that requires, in addition to IV antibiotics, the immediate decompression of the urinary tract above the point of obstruction.

A patient who is allergic to trimethoprim-sulfamethoxazole is seen in the office with dysuria and suprapubic pain. She has had urinary tract infections in the past. She is treated with ciprofloxacin. What is the mechanism of action of this antibiotic? A. It inhibits dihydrofolate reductase B. It inhibits DNA-dependent RNA polymerase C. It inhibits protein synthesis by binding to the 30s ribosomal subunit D. It inhibits protein synthesis by binding to the 50s ribosomal subunit E. It inhibits topoisomerase II (DNA gyrase)

E. It inhibits topoisomerase II (DNA gyrase) Ciprofloxacin and norfloxacin belong to a category of antibiotics called the fluoroquinolones. They are bactericidal and work by inhibiting topoisomerase II (DNA gyrase). They are effective against gram-negative rods and are oral agents effective against Pseudomonas. Ciprofloxacin is effective for treating urinary tract infections, gonorrhea, diarrheal diseases, and soft tissue infections. It is also used to treat Pseudomonas infections in patients with cystic fibrosis. Inhibition of dihydrofolate reductase (choice A) is the mechanism of action of trimethoprim, which is typically used in combination with sulfonamides (trimethoprim-sulfamethoxazole). Sulfonamides inhibit an earlier step in folate synthesis (dihydropteroate synthase), so the combination with trimethoprim is an effective one-two punch. Trimethoprim-sulfa is used in the treatment of Shigella, Salmonella, recurrent urinary tract infections, community-acquired methicillin resistant staph, and Pneumocystis carinii pneumonia.

A 63-year-old man who weighs 65 kg is in his second postoperative day after an abdominoperineal resection for cancer of the rectum. An indwelling Foley catheter was left in place after surgery. The nurses are concerned because, even though his vital signs have been stable, his urinary output in the past 2 hours has been zero. In the preceding 3 hours, they had collected 56 mL, 73 mL, and 61 mL, respectively. Which of the following is the most likely diagnosis? A. Acute renal failure B. Damage to the bladder during the operation C. Damage to the ureters during the operation D. Dehydration E. Plugged or kinked catheter

E. Plugged or kinked catheter In the presence of normal perfusion pressure, biologic problems do not suddenly drive the urinary output from normal to zero. Such a change is invariably caused by a mechanical problem.

A 78-year-old woman with a history of renal calculi and recurrent urinary tract infections presents with fever, chills, leukocytosis, and cloudy urine that has a pH of 8.2. A urine culture grows a lactose-negative, urease-positive, gram-negative rod. Which of the following microorganisms is most likely responsible for her infection? A. Candida albicans B. Enterococcus faecalis C. Escherichia coli D. Klebsiella pneumoniae E. Proteus mirabilis

E. Proteus mirabilis Proteus mirabilis is a gram-negative rod that is a member of the family Enterobacteriaceae. It is lactose-negative and contains urease, which splits urea, raising the pH of the urine to create a more hospitable environment for the bacterium. Patients who have stones are at increased risk for Proteus because the organism is able to hide in the stones, and patients who have Proteusare more likely to get stones because the increased urinary pH contributes to their formation. Proteus-producing organisms may cause struvite stones, which are also described as being staghorn calculi.

A 43-year-old man seeks help for erectile dysfunction. He gives a history of impotence of sudden onset, which was first manifested at a time of significant marital stress. There is no history of perineal trauma or pelvic surgery. Outpatient testing reveals that he has normal testosterone levels and presence of nocturnal penile tumescence (nocturnal erections during REM sleep).Which of the following is the most likely diagnosis? A. Hyperprolactinemia B. Impotence due to arterial insufficiency C. Impotence due to venous leak D. Neurogenic impotence E. Psychogenic impotence

E. Psychogenic impotence The sudden onset and the continued presence of nocturnal erections point to a psychogenic problem.

A 5-month-old male infant has a urine output of less than 0.1 mL/kg/hr shortly after undergoing major surgery. On examination, he has generalized edema. His blood pressure is 94/48 mm Hg, pulse is 140/min, and respirations are 20/min. His blood urea nitrogen is 38 mg/dL, and serum creatinine is 1.4 mg/dL. Initial urinalysis shows a specific gravity of 1.018 and 2+ protein. Microscopic examination of the urine sample reveals 1 WBC per high-power field (HPF), 18 RBCs per HPF, and 5 granular casts per HPF. His fractional excretion of sodium is 3.2%. Which of the following is the most appropriate next step in diagnosis? A. CT of the abdomen and pelvis B. Cystourethrography C. Intravenous pyelography D. Renal biopsy E. Renal ultrasonography

E. Renal ultrasonography This infant developed acute renal failure (ARF) in the immediate post-op period, as manifested by the increase in blood urea nitrogen and serum creatinine and the decrease in urine output. ARF can be classified into prerenal, renal, and postrenal. Prerenal causes include hypovolemia secondary to severe dehydration, hemorrhage, and hypotension secondary to shock. Renal causes include acute tubular necrosis (ATN), parenchymal disorders (e.g., glomerulonephritis), and vascular disorders (e.g., renal artery thrombosis or renal vein thrombosis). Postrenal causes include ureteral or urethral obstruction. This infant most likely has ATN, which is caused by ischemic or toxic injury to the nephrons. Ischemia can be caused by hypovolemia, low cardiac output states, or renal vasoconstriction. Toxins include contrast agents, antibiotics, uric acid, and myoglobin. ATN is characterized by mild proteinuria, microscopic hematuria, and the presence of coarse granular casts in the urine. A fractional excretion of sodium greater than 2% (or 2.5% in neonates) is consistent with renal causes of ARF.

A 6-year-old boy is brought to the emergency department because of headaches. The parents report that he has passed very little urine since yesterday, and that his urine color was smoky brown. He was previously well, except for a skin infection 4 weeks ago. His temperature is 37.2° C (99.0° F), blood pressure 148/102 mm Hg, pulse 98/min, and respirations 22/min. Examination shows periorbital edema, normal lungs and abdomen, normal cardiac rhythm, and a third heart sound. After controlling his blood pressure with nifedipine, the patient is admitted for management. Laboratory studies show: CBC: Hemoglobin 11.4 WBC 8.4x10^9 Differential: Polymorphs 48%, lymphocytes 42%, monocytes 8% Platelets 280x10^9 Serum chemistry: Sodium 136 Potassium 4.3 Chloride 106 Bicarbonate 26 Calcium 7.1 BUN 28 Creatinine 2.1 C3 complement 28 (normal 75-200) Albumin 3.9 Anti-DNase B titer 1400 U (normal 240-480( Urine: pH 6.5 Specific gravity 1.015 Glucose negative Protein moderate Blood large Red cell casts present Which of the following is the most appropriate treatment? A. Trimethoprim-sulfamethoxazole B. IV immunoglobulin C. Peritoneal dialysis D. Prednisone E. Salt restriction

E. Salt restriction The presence of oliguria (<100 mL/24 h) hematuria with dysmorphic red blood cells and red blood cell casts, elevated creatinine, low complements (C3, C4, and CH50 levels), and evidence of previous streptococcal infection (elevated anti-DNase B level) are indicative of a diagnosis of acute poststreptococcal glomerulonephritis (PSGN). The diagnosis is made clinically and renal biopsy is not required in this case for confirmation. The condition is self-limited and only supportive therapy is required. A course of penicillin therapy is given to reduce the spread of nephrogenic strains of Streptococcus, but it has no effect on the outcome of the illness. Hypertension is common in the acute phase and should be managed with salt restriction and antihypertensive agents. With this patient having headache as part of his presentation, nifedipine is a recommended agent for blood pressure management in a patient with hypertensive encephalopathy.

An elderly woman is brought to the emergency department extremely lethargic and confused. She is unable to give any history. Her temperature is 39.0°C (102.2°F), blood pressure 100/70 mm Hg, pulse 143/min, and respirations 17/min. Physical examination shows photophobia and nuchal rigidity. Her mucous membranes are moist. The heart is tachycardic and regular. Lungs are clear. The abdomen is benign. There is no edema. Laboratory studies show: Sodium 114 Potassium 3.5 Chloride 100 Bicarbonate 24 Glucose 116 BUN 17 Creatinine 1.1 Albumin 3.7 INR 1.02 Hemoglobin 12 mg/dL WBCs 27,000 Platelets 270,000 Hct 31% Urine sodium 42 mEq/L Plasma osmolality 270 mOsmol/kg Urine output 100-300 mL/hr Which of the following is the most likely etiology of this patient's hyponatremia? A. Adrenal insufficiency B. Cirrhosis C. Congestive heart failure D. Diuretics E. Syndrome of inappropriate antidiuretic hormone

E. Syndrome of inappropriate antidiuretic hormone The evaluation of hyponatremia begins with a determination of volume status. Our patient has no edema and clear lungs. She is normotensive and has a normal creatinine. It is therefore reasonable to suspect that she is euvolemic. In a euvolemic patient, the causes of hyponatremia include psychogenic polydipsia, hypothyroidism, and syndrome of inappropriate antidiuretic hormone. This patient's underlying problem is most likely bacterial meningitis. Any CNS abnormality can result in the syndrome of inappropriate antidiuretic hormone, which is characterized by an inability to produce dilute urine. Rather, the body retains free water despite euvolemia, and hyponatremia develops. Syndrome of inappropriate antidiuretic hormone should be suspected in any patient who has hyponatremia together with hypoosmolality.

A 55-year-old man has severe lower abdominal pain and suprapubic pressure. Earlier in the day, while undergoing coronary catheterization for unstable angina, he had transient hypotension and bradycardia that was successfully treated with atropine and epinephrine. He has not urinated since the procedure. He admitted to having urinary hesitancy for the past several months prior to this procedure. The nurse notes that he is tachycardic, with a regular pulse of 110/min. A review of systems taken on admission reveals nocturia and a weak urinary stream. From a historical context, which of the following is the most likely etiology of his abdominal pain? A. Cholesterol emboli syndrome B. Contrast-induced acute renal failure C. Nephrolithiasis D. Pyelonephritis E. Urinary retention

E. Urinary retention One of the most embarrassing findings on urgent abdominal CT scans is an enlarged bladder from urinary retention. The fact that the patient had symptoms of an enlarged prostate prior to admission, combined with the atropine that he received during his catheterization, placed him at risk for urinary retention. In addition to anticholinergic drugs like atropine, other medications that have anticholinergic effects (e.g., tricyclic antidepressants) may also place patients at risk for urinary retention.


Related study sets

Chapter 48: Diabetes Mellitus Lewis: Medical-Surgical Nursing, 11th Edition

View Set

Identify rational and irrational numbers

View Set

Chapter 5 T/F Review - Computer Programming

View Set

Series 66: Uniform Securities Act (Securities Registration)

View Set

Intensive vs Extensive Properties

View Set